Chapter 26: Management of Patients with Dysrhythmias and Conduction Problems

¡Supera tus tareas y exámenes ahora con Quizwiz!

A nurse enters a client's room and finds the client pulseless and unresponsive. What would be the treatment of choice for this client? A. Chemical cardioversion B. Electric cardioversion C. IV lidocaine D. Immediate CPR

D. Immediate CPR Immediate CPR is used during pulseless ventricular tachycardia and ventricular fibrillation

A client has started to exhibit dangerous PVCs in the cardiac postoperative unit. The client has been given a bolus of lidocaine and is under continuous IV infusion, but serious side effects, including hypotension during administration, could occur. What should the nurse be ready to do? A. Adjust the IV infusion. B. Prepare for defibrillation. C. Call for the doctor and just wait. D. Administer additional lidocaine.

A. Adjust the IV infusion. Call for the physician while adjusting the IV infusion to the slowest possible rate until the physician can examine the patient.

The registered nurse reviewed the patient's vital signs and noted a consistent pattern of heart rate recordings between 48 and 58 bpm over a 24-hour period of time. What medication will cause bradycardia? A. Atenolol B. Aminophylline C. Atropine D. Epinephrine

A. Atenolol Atenolol is a beta-blocker that can lower the heart rate. The other medications stimulate the sympathetic response which will increase heart rate.

he nurse explains to the patient with PACs that there are many causes, some of which are modifiable. Select all that apply. A. Atrial ischemia B. Anxiety C. Hyperkalemia D. Alcohol E. Hypoxemia F. Hypovolemia

A. Atrial ischemia B. Anxiety D. Alcohol E. Hypoxemia All of these causes stimulate or increase electrical impulses in the heart except for hyperkalemia.

The nurse cares for a client following the insertion of a permanent pacemaker. What discharge instruction(s) should the nurse review with the client? Select all that apply. A. Avoid handheld screening devices in airports B. Check pulse daily, reporting sudden slowing or increase C. Wear a medical alert, noting the presence of a pacemaker D. Avoid the usage of microwave ovens and electronic tools E. Refrain from walking through antitheft devices

A. Avoid handheld screening devices in airports B. Check pulse daily, reporting sudden slowing or increase C. Wear a medical alert, noting the presence of a pacemaker

A client is prescribed quinine for treatment of moderate depression of depolarization. Which sign indicates the drugs' effectiveness in the client's treatment? A. Decrease in cardiac contractility B. Hypertension with IV administration. C. Shortened QT segment D. Decreased QRS duration of about 40% from baseline

A. Decrease in cardiac contractility Quinine works by increasing the heart's resistance to activity and causes decreased cardiac contractility. The nurse needs to monitor the blood pressure for signs of hypotension and observe the client for signs of heart failure, chest pain, and arrhythmias.

Which of the following does the nurse recognize as the therapeutic goal of radiofrequency catheter ablation for a client with cardiac dysrhythmias? A. Destruction of errant tissue B. Stimulation of the impulse center C. Dilation of arterial blood vessels D. Reperfusion of ischemic heart tissue

A. Destruction of errant tissue The therapeutic goal of radiofrequency catheter ablation is to destroy errant tissue, in hopes of allowing impulse conduction to travel over appropriate pathways. The goal does not include dilation of blood vessels or reperfusion of heart tissue. There is no stimulation of the heart.

Elective cardioversion is similar to defibrillation except that the electrical stimulation waits to discharge until an R wave appears. What does this prevent? A. Disrupting the heart during the critical period of ventricular repolarization B. Disrupting the heart during the critical period of atrial repolarization C. Disrupting the heart during the critical period of atrial depolarization D. Disrupting the heart during the critical period of ventricular depolarization

A. Disrupting the heart during the critical period of ventricular repolarization It is similar to defibrillation. One difference is that the machine that delivers the electrical stimulation waits to discharge until it senses the appearance of an R wave. By doing so, the machine prevents disrupting the heart during the critical period of ventricular repolarization. Therefore, the other options are incorrect.

The nurse is caring for a client with atrial fibrillation. What procedure would be recommended if drug therapies did not control the dysrhythmia? A. Elective cardioversion B. Pacemaker implantation C. Maze procedure D. Defibrillation

A. Elective cardioversion Atrial fibrillation also is treated with elective cardioversion or digitalis if the ventricular rate is not too slow. Defibrillation is used for a ventricular problem. A Maze procedure is only a distractor for this question. Pacemakers are implanted for bradycardia.

A client with atrial fibrillation, who does not respond to conventional treatment measures and who is not a candidate for cardioversion, would have what procedure recommended? A. Maze procedure B. Heart transplantation C. Angiography D. Open heart surgery

A. Maze procedure Clients who are not candidates for cardioversion and fail to respond to conventional measures may be candidates for a surgical intervention referred to as the Maze procedure. Angiography, open heart surgery, and heart transplantation are not recommended surgeries for these clients.

The nurse expects to see which of the following characteristics on an ECG strip for a patient who has third-degree AV block? A. More P waves than QRS complexes B. Extended PR interval C. Shortened QRS duration. D. Atrial rate of 60 bpm or below

A. More P waves than QRS complexes There is no PR interval because there isn't any relationship between the P and R wave. No atrial impulse is conducted through the AV node; atrial and ventricular contractions are independent. With third-degree AV block, two separate impulses stimulate the heart; there is no synchrony or relationship.

Undersensing occurs as a pacemaker malfunctions. The nurse understands undersensing occurs as a result of which event? A. Pacing spike occurs at the preset level B. Loss of pacing artifact C. The complex does not follow the pacing spike D. Total absence of the pacing spike

A. Pacing spike occurs at the preset level Undersensing means that the pacing spike occurs at preset interval despite the patient's intrinsic rhythm. Loss of capture occurs when a complex does not follow a pacing spike. Loss of pacing is total absence of the pacing spike. Oversensing occurs when there is a loss of pacing artifact.

The nurse cares for a client who has developed junctional tachycardia with a heart rate (HR) of 80 bpm. Which action should the nurse complete? A. Request a digoxin level be ordered B. Prepare for emergent electrical cardioversion C. Prepare to administer IV lidocaine D. Withhold the client's oral potassium supplement

A. Request a digoxin level be ordered The nurse should request a digoxin level be obtained. Junctional tachycardia generally does not have any detrimental hemodynamic effect; it may indicate a serious underlying condition, such as digitalis toxicity, myocardial ischemia, hypokalemia, or chronic obstructive pulmonary disease (COPD). Potassium supplements do not cause junctional tachycardia. Lidocaine is indicated for the treatment of premature ventricular contractions (PVCs). Because junctional tachycardia is caused by increased automaticity, cardioversion is not an effective treatment; in fact, it causes an increase in ventricular rate.

The nurse checks the synchronizer switch before using a defibrillator to terminate ventricular fibrillation for what important reason? A. The defibrillator won't deliver a shock if the synchronizer switch is turned on. B. The shock must be synchronized with the client's T wave. C. The defibrillator won't deliver a shock if the synchronizer switch is turned off. D. The delivered shock must be synchronized with the client's QRS complex.

A. The defibrillator won't deliver a shock if the synchronizer switch is turned on. The nurse needs to check the synchronizer switch to ensure the switch is turned off. The defibrillator won't deliver a shock to the client in ventricular fibrillation if the synchronizer switch is turned on because the defibrillator needs to recognize a QRS complex when the switch is turned on. The synchronizer switch should be turned on when attempting to terminate arrhythmias that contain QRS complexes, such as rapid atrial fibrillation that's resistant to pharmacologic measures. A synchronized shock should occur with the QRS complex, not the T wave, to avoid inducing ventricular fibrillation and allow for a lower shock dose.

Which electrocardiogram (ECG) characteristic is usually seen when a client's serum potassium level is low? A. U wave B. T wave C. QT interval D. P wave

A. U wave The U wave is an ECG waveform characteristic that may reflect Purkinje fiber repolarization. It is usually seen when a client's serum potassium level is low. The T wave is an ECG characteristic reflecting repolarization of the ventricles. It may become tall or "peaked" if a client's serum potassium level is high. The P wave is an ECG characteristic reflecting conduction of an electrical impulse through the atria. The QT interval is an ECG characteristic reflecting the time from ventricular depolarization to repolarization.

The nurse is working on a monitored unit assessing the cardiac monitor rhythms. Which waveform pattern needs attention first? A. Ventricular fibrillation B. Supraventricular tachycardia C. Sustained asystole D. Atrial fibrillation

A. Ventricular fibrillation Ventricular fibrillation is called the rhythm of a dying heart. It is the rhythm that needs attention first because there is no cardiac output, and it is an indication for CPR and immediate defibrillation. Sustained asystole either is from death, or the client is off of the cardiac monitor. Supraventricular tachycardia and atrial fibrillation is monitored and reported to the physician but is not addressed first.

A 1-minute electrocardiogram (ECG) tracing of a client with a regular heart rate reveals 25 small, square boxes within an RR interval. The nurse correctly identifies the client heart rate as A. 70 bpm. B. 60 bpm. C. 80 bpm. D. 100 bpm.

B. 60 bpm. A client's HR can be obtained from the electrocardiogram (ECG) tracing by several methods. A 1-minute strip contains 300 large boxes and 1500 small boxes. Therefore, an easy and accurate method of determining heart rate with a regular rhythm is to count the number of small boxes within an RR interval and divide by 1,500. In this instance, 1,500/25 = 60.

The nurse is assigned to care for several clients admitted to a telemetry unit. Which clients should the nurse assess first? A. A client returned from an electrophysiology procedure 2 hours ago, reporting constipation B. A client whose implantable cardioverter defibrillator (ICD) fired twice on the prior shift, requiring amiodarone IV C. A client who received elective cardioversion 1 hour ago and whose heart rate (HR) is 115 bpm D. A client diagnosed with new onset of atrial fibrillation, requiring scheduled IV diltiazem

B. A client whose implantable cardioverter defibrillator (ICD) fired twice on the prior shift, requiring amiodarone IV The client's implantable cardioverter defibrillator (ICD) that has fired on the previous shift should be seen first. This client is in need of antidysrhythmic medication, which is the priority intervention. The remaining clients should be seen after this client and are in no acute distress.

A nurse is caring for a client who has had an automatic cardiac defibrillator implanted. What instructions should the nurse provide to the client? A. Use digital cellular telephones. B. Avoid devices with a magnetic field. C. Avoid driving for at least 3 months. D. Avoid using microwave ovens.

B. Avoid devices with a magnetic field. The nurse should instruct the client to avoid devices with a magnetic field. Examples include magnetic resonance imaging devices, extracorporeal shock wave lithotripsy machines, electrocautery and diathermy devices, peripheral nerve stimulators, large industrial electrical motors, and arc welding equipment. Electrical signals from digital cellular telephones may mimic an abnormal heart rhythm, activating the automatic implanted cardiac defibrillator. The nurse should instruct the client to use analog cellular telephones and to restrict driving for at least 6 months. Microwave ovens have shields or are grounded, making them safe for use by clients with an automatic implanted cardiac defibrillator.

The nurse receives a client from the postanesthesia unit with a new onset of sinus tachycardia and a heart rate of 118 beats per minute. To which reasons does the nurse relate the increased heart rate? Select all that apply. A. Hypoglycemia B. Fever C. Sleep D. Anxiety E. Blood loss

B. Fever D. Anxiety E. Blood loss Fever, blood loss, and anxiety can be related to tachycardia. Sleep and hypoglycemia are contributing factors to bradycardia.

A 43-year-old male came into the emergency department where you practice nursing and was diagnosed with atrial fibrillation. It's now 48 hours since his admittance and the dysrhythmia persists. Which of the following medications will the client's healthcare provider most likely order? A. Warfarin (Coumadin) B. Heparin C. Flecainide (Tambocor) D. Dabigatran (Pradaxa)

B. Heparin Heparin is generally prescribed initially if the dysrhythmia persists longer than 48 hours.

The nurse understands that asystole can be caused by several conditions. Select all that apply. A. Alkalosis B. Hypoxia C. Hypothermia D. Hypovolemia E. Acidosis

B. Hypoxia C. Hypothermia D. Hypovolemia E. Acidosis Ventricular asystole is treated the same as pulseless electrical activity (PEA), focusing on high-quality cardiopulmonary resuscitation (CPR) with minimal interruptions and identifying underlying and contributing factors. The key to successful treatment is a rapid assessment to identify a possible cause, which is known as the "Hs and Ts": hypoxia, hypovolemia, hydrogen ion (acid/base imbalance), hypo- or hyperglycemia, hypo- or hyperkalemia, hyperthermia, trauma, toxins, tamponade (cardiac), tension pneumothorax, or thrombus (coronary or pulmonary).

The nurse enters the client's room and finds the client pulseless and unresponsive. What would be the treatment of choice for this client? A. IV lidocaine B. Immediate defibrillation C. Chemical cardioversion D. Electric cardioversion

B. Immediate defibrillation Defibrillation is used during pulseless ventricular tachycardia and ventricular fibrillation.

The nurse is attempting to determine the ventricular rate and rhythm of a patient's telemetry strip. What should the nurse examine to determine this part of the analysis? A. TP interval B. RR interval C. PP interval D. QT interval

B. RR interval The rhythm is often identified at the same time the rate is determined. The RR interval is used to determine ventricular rhythm and the PP interval to determine atrial rhythm. If the intervals are the same or if the difference between the intervals is less than 0.8 seconds throughout the strip, the rhythm is called regular. If the intervals are different, the rhythm is called irregular.

Which dysrhythmia is common in older clients? A. Ventricular tachycardia B. Sinus bradycardia C. Sinus tachycardia D. Atrial fibrillation

B. Sinus bradycardia Sinus bradycardia is a common dysrhythmia in older clients. Sinus tachycardia, atrial fibrillation, and ventricular tachycardia are not common dysrhythmias in older clients.

The nurse knows which heart rhythm occurs when the atrial and ventricular rhythms are both regular, but independent of each other? A. Second-degree heart block B. Third-degree atrioventricular (AV) heart block C. Asystole D. First-degree AV block

B. Third-degree atrioventricular (AV) heart block In third-degree AV heart block there is no relationship or synchrony between the atrial and ventricular contraction. Each is beating at its own inherent rate and is independent of each other, thus the cardiac output is affected. Second-degree AV block occurs when only some of the atrial impulses are conducted through the AV node into the ventricles. First-degree AV block occurs when atrial conduction is delayed through the AV node, resulting in a prolonged PR interval. During asystole, there is no electrical activity.

A patient who had a myocardial infarction is experiencing severe chest pain and alerts the nurse. The nurse begins the assessment but suddenly the patient becomes unresponsive, no pulse, with the monitor showing a rapid, disorganized ventricular rhythm. What does the nurse interpret this rhythm to be?\ A. Ventricular tachycardia B. Ventricular fibrillation C. Atrial fibrillation D. Third-degree heart block

B. Ventricular fibrillation The most common dysrhythmia in patients with cardiac arrest is ventricular fibrillation, which is a rapid, disorganized ventricular rhythm that causes ineffective quivering of the ventricles. No atrial activity is seen on the ECG. The most common cause of ventricular fibrillation is coronary artery disease and resulting acute myocardial infarction. Ventricular fibrillation is always characterized by the absence of an audible heartbeat, a palpable pulse, and respirations.

A client with a history of mitral stenosis is admitted to the intensive care unit (ICU) with the abrupt onset of atrial fibrillation. The client's heart rate ranges from 120 to 140 bpm. The nurse recognizes that interventions are implemented to prevent the development of A. myocardial infarction. B. embolic stroke. C. heart failure. D. renal failure.

B. embolic stroke. Intervention is implemented to prevent the development of an embolic event/stroke. Clients with a history of previous stroke, transient ischemic attack (TIA), embolic event, mitral stenosis, or prosthetic heart valve and who develop atrial fibrillation are at significant risk of developing an embolic stroke. Antithrombotic therapy is indicated for all clients with atrial fibrillation, especially those at risk of an embolic event, such as a stroke, and it is the only therapy that decreases cardiovascular mortality. These client are often placed on warfarin, in contrast to clients who have no risk factors, and who are often prescribed 81 to 325 mg of aspirin daily.

A client has had an automatic implanted cardioverter defibrillator (AICD) implanted. The nurse explains the purpose and benefits of the AICD to the client's family. The least likely reason for the client's AICD implantation is that the client: A. experiences recurrent episodes of ventricular tachycardia. B. experiences recurrent episodes of atrial flutter. C. has survived at least one episode of cardiac arrest with ventricular origin. D. is at risk for death due to structural cardiac disease with poor ventricular function.

B. experiences recurrent episodes of atrial flutter. AICDs are used for dysrhythmias of a ventricular nature. It is not about cardiac arrest. It is not about ventricular tachycardia. It is not about structural cardiac disease.

A client is unconscious on arrival to the emergency department. The nurse in the emergency department identifies that the client has a permanent pacemaker due to which characteristic? A. Vibration under the skin B. Quality of the pulse C. "Spike" on the rhythm strip D. Scar on the chest

C. "Spike" on the rhythm strip Confirmation that the client has a permanent pacemaker is the characteristic "spike" identified by a thin, straight stroke on the rhythm strip. The scar on the chest is suggestive of pacer implantation but not definitive. There should be no change in pulse quality, and no vibration under the skin.

The nurse is caring for a client scheduled for a transesophageal echocardiogram with a diagnosis of atrial fibrillation. The client's spouse asks the nurse to explain the purpose of the test. What is the nurse's best response? A. "This test will let the doctor know if the client is at risk for hypotension." B. "This test will show the specific area causing the atrial fibrillation and what can be done to stop it." C. "This test will show any blood clots in the heart, and help us determine if it is safe to do a cardioversion." D. "This test will show if the client needs a cardiac catheterization."

C. "This test will show any blood clots in the heart, and help us determine if it is safe to do a cardioversion." The transesophageal echocardiogram will show if the client has blood clots and help determine if it is safe to use cardioversion. The transesophageal echocardiogram does not indicate which area is causing the atrial fibrillation or the need to evaluate coronary arteries, as with a cardiac catheterization. Hypotension is diagnosed with blood pressure readings.

A client has had a pacemaker implanted and the nurse will begin client education upon the client becoming alert. Which postimplantation instructions must be provided to the client with a permanent pacemaker? A. Delay activities such as swimming and bowling for at least 3 weeks. B. Keep the arm on the side of the pacemaker higher than the head. C. Avoid sources of electrical interference. D. Keep moving the arm on the side where the pacemaker is inserted.

C. Avoid sources of electrical interference. The nurse must instruct the client with a permanent pacemaker to avoid sources of electrical interference, such as MRI devices, large industrial motors, peripheral nerve stimulators, etc.

The nurse knows that a pacemaker is the treatment of choice for what cardiac dysrhythmia? A. Supraventricular tachycardia B. Ventricular fibrillation C. Complete heart block D. Atrial flutter

C. Complete heart block Pacemaker insertion is the treatment for complete heart block. Treatments for supraventricular tachycardia are: valsalva maneuver, unilateral carotid massage, immersion of face in ice water, administration of IV adenosine, cardioversion, and radiofrequency ablation. Cardioversion and drug therapy are used for the treatment of atrial flutter. Treatment for ventricular fibrillation is defibrillation preceded by or followed with epinephrine.

The nurse recognizes which as being true of cardioversion? A. Defibrillator should be set in the non-synchronous mode so the nurse can hit the button at the right time. B. Electrical impulse can be discharged during the T wave. C. Defibrillator should be set to deliver a shock during the QRS complex. D. Amount of voltage used should exceed 400 watts/second.

C. Defibrillator should be set to deliver a shock during the QRS complex. Cardioversion involves the delivery of a "timed" electrical current. The defibrillator is set to synchronize with the ECG and deliver the impulse during the QRS complex. The synchronization prevents the discharge from occurring during the vulnerable period of repolarization (T wave), which could result in VT or ventricular fibrillation.

Which of the following medications does the nurse anticipate administering to a client preparing for cardioversion? A. Enalapril B. Digoxin C. Diazepam D. Atropine

C. Diazepam Prior to cardioversion, cardiac medications are held, and the client is sedated with a medication such as diazepam.

The nurse is assessing a client with mitral regurgitation. The nurse expects to note what finding in this client? A. Dizziness, syncope, and palpitations B. Dry cough, wheezing, and hemoptysis C. Dyspnea, fatigue, and weakness D. Orthopnea, angina, and pulmonary edema

C. Dyspnea, fatigue, and weakness Chronic mitral regurgitation is often asymptomatic, but acute mitral regurgitation can cause dyspnea, fatigue, and weakness. Dizziness, syncope, and palpitations are usually symptoms of mitral valve prolapse. Orthopnea, angina, and pulmonary edema are more likely with aortic stenosis. Dry cough, wheezing, and hemoptysis are more likely with mitral stenosis.

What is the treatment of choice for ventricular fibrillation? A. Pacemaker B. Implanted defibrillator C. Immediate bystander CPR D. Atropine

C. Immediate bystander CPR The treatment of choice for ventricular fibrillation is immediate bystander cardiopulmonary resuscitation (CPR), defibrillation as soon as possible, and activation of emergency services.

A client is treated in the intensive care unit (ICU) following an acute myocardial infarction (MI). During the nursing assessment, the client reports shortness of breath and chest pain. In addition, the client's blood pressure (BP) is 100/60 mm Hg with a heart rate (HR) of 53 bpm, and the electrocardiogram (ECG) tracing shows more P waves than QRS complexes. Which action should the nurse complete first? A. Administer 1 mg of IV atropine B. Obtain a 12-lead ECG C. Initiate transcutaneous pacing D. Prepare for defibrillation

C. Initiate transcutaneous pacing The client is experiencing a third-degree heart block. Transcutaneous pacing should be implemented first. A permanent pacemaker may be indicated if the block continues. Defibrillation is not indicated; third-degree heart block does not respond to atropine; a 12-lead ECG may be obtained, but is not completed first.

A nurse should obtain serum levels of which electrolytes in a client with frequent episodes of ventricular tachycardia? A. Calcium and magnesium B. Potassium and calcium C. Magnesium and potassium D. Potassium and sodium

C. Magnesium and potassium Hypomagnesemia as well as hypokalemia and hyperkalemia are common causes of ventricular tachycardia. Calcium imbalances cause changes in the QT interval and ST segment. Alterations in sodium level don't cause rhythm disturbances.

A nurse is caring for a client who is on a continuous cardiac monitor. When evaluating the client's rhythm strip, the nurse notes that the QRS interval has increased from 0.08 second to 0.14 second. Based on this finding, the nurse should withhold continued administration of which drug? A. Propafenone B. Verapamil C. Procainamide D. Metoprolol

C. Procainamide Procainamide may cause an increased QRS complexes and QT intervals. If the QRS duration increases by more than 50%, then the nurse should withhold the drug and notify the physician of her finding. Metoprolol may cause increased PR interval and bradycardia. Propafenone and verapamil may cause bradycardia and atrioventricular blocks.

The nurse is preparing to administer adenosine for a patient diagnosed with atrial flutter. How should the nurse administer this medication? A. The medication is followed by a slow saline flush. B. The medication is followed by a rapid lactated Ringer's (LR) flush C. The dose is administered rapid IV push. D. The dose is administered slow IV push.

C. The dose is administered rapid IV push. The dose is administered by rapid IV push (1 to 2 seconds), followed with a rapid saline flush. LR is not used during administration of this medication.

The nurse knows that synchronization of cardioversion prevents the discharge from occurring during the vulnerable period of which of the following? A. Atrial depolarization B. Purkinje fiber repolarization C. Ventricular repolarization D. Ventricular depolarization

C. Ventricular repolarization The synchronization of cardioversion prevents the discharge from occurring during the vulnerable period of repolarization (T wave), which could result in ventricular tachycardia (VT) or ventricular fibrillation. The P wave represents atrial depolarization. The QRS complex represents ventricular depolarization. The U wave represents repolarization of the Purkinje fibers.

A client experiences a faster-than-normal heart rate when drinking more than two cups of coffee in the morning. What does the nurse identify on the electrocardigram as an indicator of sinus tachycardia? A. Q wave of 0.04 seconds B. PR interval of 0.1 seconds C. heart rate of 118 bpm D. QRS duration of 0.16 seconds

C. heart rate of 118 bpm The sinus node creates an impulse at a faster-than-normal rate. The PR interval of 0.1 seconds, QRS duration of 0.16 seconds and Q wave of 0.04 seconds are consistent with a normal sinus rhythm. Sinus tachycardia occurs when the heart rate is over 100 bpm

The nurse is educating the client about a transvenous pacemaker. What is the best statement to explain why the client will have a transvenous pacemaker? A. "A transvenous pacemaker is used in place of a transarterial pacemaker." B. "A transvenous pacemaker is used for a ventricular tachydysrhythmias." C. "A transvenous pacemaker is a permanent pacemaker that is asynchronous." D. "A transvenous pacemaker is used to manage transient bradydysrhythmias like those that occur during acute MIs."

D. "A transvenous pacemaker is used to manage transient bradydysrhythmias like those that occur during acute MIs." A transvenous pacemaker is a temporary pulse-generating device that sometimes is necessary to manage transient bradydysrhythmias such as those that occur during acute MIs or after coronary artery bypass graft surgery, or to override tachydysrhythmias. The transvenous pacemaker is a temporary pacemaker introduced into the venous system, not the arterial system. The pacemaker does not manage ventricular dysrhythmias.

The nurse is working with a client with a new onset of atrial fibrillation during a three-month follow-up visit. The healthcare provider is planning a cardioversion, and the client asks the nurse why there is a wait for the treatment. What is the best response by the nurse? A. "There is a long list of clients in line to be cardioverted." B. "We have to allow your heart to rest for a few weeks before it is stressed by the cardioversion." C. "The doctor wants to see if your heart will switch back to its normal rhythm by itself." D. "Your atrial chambers may contain blood clots now, so you must take an anticoagulant for a few weeks before the cardioversion."

D. "Your atrial chambers may contain blood clots now, so you must take an anticoagulant for a few weeks before the cardioversion." Because of the high risk of embolization of atrial thrombi, cardioversion of atrial fibrillation that has lasted longer than 48 hours should be avoided unless the client has received warfarin for at least 3 to 4 weeks prior to cardioversion. The doctor will not wait for a change in rhythm. Resting the heart will not change the rhythm. There is no delay but safer for the clots to be dissolved with the anticoagulant.

The nurse is defibrillating a patient in ventricular fibrillation with paddles on a monophasic defibrillator. How much paddle pressure should the nurse apply when defibrillating? A. 5 to 10 lbs B. 15 to 20 lbs C. 5 to 10 lbs D. 20 to 25 lbs

D. 20 to 25 lbs When using paddles, 20 to 25 lb of pressure must be used in order to ensure good skin contact.

A client is scheduled for an elective electrical cardioversion for a sustained dysrhythmia lasting for 24 hours. Which intervention is necessary for the nurse to implement prior to the procedure? A. Maintain the client on NPO status for 8 hours prior to the procedure B. Administer anticoagulant therapy as prescribed prior to the procedure C. Administer the prescribed digitalis to the client before the scheduled procedure D. Administer moderate sedation IV and analgesic medication as prescribed

D. Administer moderate sedation IV and analgesic medication as prescribed Before an elective cardioversion, the client should receive moderate sedation IV as well as an analgesic medication or anesthesia. In contrast, in emergent situations, the client may not be premedicated. Digoxin is usually withheld for 48 hours before cardioversion to ensure the resumption of sinus rhythm with normal conduction. If the cardioversion is elective and the dysrhythmia has lasted longer than 48 hours, anticoagulation performed for a few weeks before cardioversion may be indicated. The client is instructed not to eat or drink for at least 4 hours before the procedure.

Two clients in cardiac rehabilitation are discussing the differences between scheduled cardioversion and unexpected defibrillation. Which statement does the nurse note is not correct? A. Both procedures sedate the clients. B. Machine determines when electrical energy is delivered. C. Both used to eliminate ventricular dysrhythmias. D. Cardioversion uses more electrical energy than defibrillation.

D. Cardioversion uses more electrical energy than defibrillation. Cardioversion uses less electrical energy (50 to 100 joules) than defibrillation (200 to 360 joules). All of the other statements are correct.

A patient has had several episodes of recurrent tachydysrhythmias over the last 5 months and medication therapy has not been effective. What procedure should the nurse prepare the patient for? A. Insertion of a permanent pacemaker B. Insertion of an ICD C. Maze procedure D. Catheter ablation therapy

D. Catheter ablation therapy Catheter ablation destroys specific cells that are the cause or central conduction route of a tachydysrhythmia. It is performed with or after an electrophysiology study. Usual indications for ablation are atrioventricular nodal reentry tachycardia, a recurrent atrial dysrhythmia (especially atrial fibrillation), or ventricular tachycardia unresponsive to previous therapy (or for which the therapy produced significant side effects).

The nurse analyzes the electrocardiogram (ECG) strip of a stable patient admitted to the telemetry unit. The client's ECG strip demonstrates PR intervals that measure 0.24 seconds. What is the nurse's most appropriate action? A. Instruct the client to bear down as if having a bowel movement B. Apply oxygen via nasal cannula and obtain a 12-lead ECG C. Notify the client's primary care provider of the findings D. Document the findings and continue to monitor the patient

D. Document the findings and continue to monitor the patient The client's electrocardiogram (ECG) tracing indicates a first-degree atrioventricular (AV) block. First-degree AV block rarely causes any hemodynamic effect; the other blocks may result in decreased heart rate, causing a decrease in perfusion to vital organs, such as the brain, heart, kidneys, lungs, and skin. The most appropriate action by the nurse is to document the findings and continue to monitor the client.

The nurse and student nurse are observing a cardioversion procedure completed by a physician. At which time is the nurse most correct to identify to the student when the electrical current will be initiated? A. During repolarization of the heart B. During stimulation of the SA node C. During the QRS complex D. During ventricular depolarization

D. During ventricular depolarization The electrical current is initiated at the R wave when ventricular depolarization occurs. The electrical current completely depolarizes the entire myocardium with the goal of restoring the normal pacemaker of the heart. The other options focus on an incorrect timing that will not restore the normal electrical conduction.

Which diagnostic study best evaluates different medications ability to restore normal heart rhythm? A. Electrocardiogram (ECG) B. Elective electrical cardioversion C. Echocardiogram D. Electrophysiology study

D. Electrophysiology study An electrophysiology study is a procedure that enables the physician to examine the electrical activity of the heart, produce actual dysrhythmias, and determine the best method for care. Cardioversion uses synchronized electricity to change the rhythm pattern. Electrocardiogram and echocardiograms provide diagnostic information.

A client receives a pacemaker to treat a recurring arrhythmia. When monitoring the cardiac rhythm strip, the nurse observes extra pacemaker spikes that don't precede a beat. Which condition should the nurse suspect? A. Asystole B. Failure to sense C. Failure to pace D. Failure to capture

D. Failure to capture Extra pacemaker spikes that don't precede a beat may indicate failure to capture, in which the pacemaker fires but the heart doesn't conduct the beat. In failure to pace, the pacemaker doesn't fire when it should; this causes hypotension and other signs of low cardiac output accompanied by bradycardia or a heart rate slower than the pacemaker's preset rate. In failure to sense, the pacemaker can't sense the client's intrinsic heartbeat; on the rhythm strip, spikes may fall on T waves, or they may fall regularly but at points at which they shouldn't appear. An absent heart rate or rhythm, reflected as a flat line on the rhythm strip, characterizes asystole.

The nurse is administering propanolol to a client on a telemetry unit. What will the nurse monitor the client for? A. bleeding B. change in level of consciousness C. tachycardia D. heart block

D. heart block Propanolol and other beta blockers can interfere with conduction and the client should be evaluated for heart block. Propanolol has a desired effect of lowering heart rate, not tachycardia. Bleeding and change in consciousness level are not common side effects of propanolol.

The nurse is assessing vital signs in a patient with a permanent pacemaker. What should the nurse document about the pacemaker? A. Date and time of insertion B. Model number C. Location of the generator D. Pacer rate

D. Pacer rate After a permanent pacemaker is inserted, the patient's heart rate and rhythm are monitored by ECG.

A patient is 2 days postoperative after having a permanent pacemaker inserted. The nurse observes that the patient is having continuous hiccups as the patient states, "I thought this was normal." What does the nurse understand is occurring with this patient? A. Faulty generator B. Sensitivity is too low C. Fracture of the lead wire D. Lead wire dislodgement

D. Lead wire dislodgement Phrenic nerve, diaphragmatic (hiccuping may be a sign), or skeletal muscle stimulation may occur if the lead is dislocated or if the delivered energy (mA) is set high. The occurrence of this complication is avoided by testing during device implantation.

The nurse, along with a nursing student, is caring for a client who is admitted with a fractured pacemaker lead related to Twiddler syndrome. The student asks for information about Twiddler syndrome. The appropriate response by the nurse is which of the following? A. "It has to do with a defective lead wire produced by a company named Twiddler." B. "A Dr. Twiddler discovered that too much arm movement will cause the lead to fracture." C. "The lead wire breaks and causes symptoms related to lack of pacing; the name has nothing to do with the cause." D. "The client twiddles with or manipulates the generator or wires, causing the lead to fracture."

D. "The client twiddles with or manipulates the generator or wires, causing the lead to fracture." Twiddler syndrome may occur when the client manipulates the generator, causing lead dislodgement or fracture of the lead.

A client's Holter monitor strip reveals a heart rate with normal conduction but with a rate consistently above 105 beats/minute. What type of dysrhythmia would the cardiologist likely diagnose? A. supraventricular bradycardia B. supraventricular tachycardia C. sinus bradycardia D. sinus tachycardia

D. sinus tachycardia Sinus tachycardia is a dysrhythmia that proceeds normally through the conduction pathway but at a faster than usual rate (100 to 150 beats/minute).

During electrical cardioversion, the defibrillator is set to synchronize with the electrocardiogram (ECG) so that the electrical impulse discharges during A. ventricular repolarization. B. atrial depolarization. C. the QT interval. D. ventricular depolarization.

D. ventricular depolarization. In cardioversion, the defibrillator is set to synchronize with the electrocardiogram (ECG) on the cardiac monitor so that the electrical impulse discharges during ventricular depolarization.

A nurse has provided discharge instructions to a client who received an implantable cardioverter defibrillator (ICD). Which statement, made by the client, indicates the need for further teaching? A. "I need to take a cardiopulmonary resuscitation (CPR) class now that I have an ICD." B. "I should tell close friends and family members that I have an ICD." C. "I will document the date and time if my ICD fires." D. "I can play golf with my son in about 2 or 3 weeks."

A. "I need to take a cardiopulmonary resuscitation (CPR) class now that I have an ICD." The client does not need to take a CPR class. However, it is recommended that the family members and friends of a client who has an ICD learn CPR. The other statements indicate that the nurse's teaching was effective.

The nurse is caring for a client who is being discharged after insertion of a permanent pacemaker. Which question by the client indicates a need for clarification? A. "I should ask for a handheld device search when I go through airport security." B. "I should avoid contact sports." C. "I should avoid large magnetic fields, such as an MRI machine or large motors." D. "I'll watch the incision for swelling or redness and will report if either occurs." SUBMIT ANSWER

A. "I should ask for a handheld device search when I go through airport security." At security gates at airports, government buildings, or other secured areas, the client with a permanent pacemaker should show a pacemaker ID card and request a hand (not handheld device) search. The client should obtain and carry a physician's letter about this requirement.

A nurse is performing discharge teaching with a client who has an implantable cardioverter defibrillator (ICD) placed. Which client statement indicates effective teaching? A. "I'll keep a log of each time my ICD discharges." B. "I need to stay at least 10 inches away from the microwave." C. "I can't wait to get back to my football league." D. "I have an appointment for magnetic resonance imaging of my knee scheduled for next week."

A. "I'll keep a log of each time my ICD discharges." The client stating that he should keep a log of all ICD discharges indicates effective teaching. This log helps the client and physician identify activities that may cause the arrhythmias that make the ICD discharge. He should also record the events right before the discharge. Clients with ICDs should avoid contact sports such as football. They must also avoid magnetic fields, which could permanently damage the ICD. Household appliances don't interfere with the ICD.

The nurse cares for a 56-year-old client who received an implantable cardioverter defibrillator (ICD) 2 days prior. The client tells the nurse "My wife and I can never have sex again now that I have this ICD." What is the nurse best response by the nurse? A. "You seem apprehensive about resuming sexual activity." B. "Sex is permitted following the implantation of an ICD." C. "I will be sure to share your concerns with the physician." D. "You really should speak to your wife about your concerns."

A. "You seem apprehensive about resuming sexual activity." The client treated with an electronic device experiences not only lifestyle and physical changes but also emotional changes. At different times during the healing process, the client may feel angry, depressed, fearful, anxious, or a combination of these emotions. It is imperative for the nurse to observe the client's response to the device and provide the client and family members with emotional support and teaching as indicated. Identifying that the client appears apprehensive about resuming sexual activity acknowledges the client's concerns while allowing for further discussion. The remaining responses ignore the client's feelings and do not facilitate an ongoing conversation or explore the client's concern.

The nurse would expect which of the following atrial rates to correlate with an idioventricular rhythm? A. 20 to 40 B. 100 to 250 C. Not measurable D. 220 to 350

A. 20 to 40 The rate is 20 to 40. If the rate exceeds 40, the rhythm is known as accelerated idioventricular rhythm (AIVR). The rate is not measurable in asystole. Ventricular tachycardia has a rate of 100 to 250 per minute. Atrial flutter has a rate of 220 to 350.

A client presents to the emergency department via ambulance with a heart rate of 210 beats/minute and a sawtooth waveform pattern per cardiac monitor. The nurse is most correct to alert the medical team of the presence of a client with which disorder? A. Atrial flutter B. Asystole C. Premature ventricular contraction D. Ventricular fibrillation

A. Atrial flutter Atrial flutter is a disorder in which a single atrial impulse outside the SA node causes the atria to contract at an exceedingly rapid rate. The atrioventricular (AV) node conducts only some impulses to the ventricle, resulting in a ventricular rate slower than the atrial rate, thus forming a sawtooth pattern on the heart monitor. Asystole is the absence of cardiac function and can indicate death. Premature ventricular contraction indicates an early electric impulse and does not necessarily produce an exceedingly rapid heart rate. Ventricular fibrillation is the inefficient quivering of the ventricles and indicative of a dying heart.

The nurse is assessing a patient with a probable diagnosis of first-degree AV block. The nurse is aware that this dysrhythmia is evident on an ECG strip by what indication? A. Delayed conduction, producing a prolonged PR interval B. P waves hidden within the QRS complex C. Irregular rhythm D. Variable heart rate, usually fewer than 90 bpm

A. Delayed conduction, producing a prolonged PR interval First-degree AV block may occur without an underlying pathophysiology, or it can result from medications or conditions that increase parasympathetic tone. It occurs when atrial conduction is delayed through the AV node, resulting in a prolonged PR interval.

A nurse evaluates a client with a temporary pacemaker. The client's ECG tracing shows each P wave followed by the pacing spike. What is the nurse's best response? A. Document the findings and continue to monitor the client B. Check the security of all connections and increase the milliamperage C. Obtain a 12-lead ECG and a portable chest x-ray D. Reposition the extremity and turn the client to left side

A. Document the findings and continue to monitor the client Capture is a term used to denote that the appropriate complex is followed by the pacing spike. In this instance, the patient's temporary pacemaker is functioning appropriately; all Ps wave followed by an atrial pacing spike. The nurse should document the findings and continue to monitor the client. Repositioning the client, placing the client on the left side, checking the security of all connections, and increasing the milliamperage are nursing interventions used when the pacemaker has a loss of capture. Obtaining a 12-lead ECG and chest x-ray are indicated when there is a loss of pacing-total absence of pacing spikes or when there is a change in pacing QRS shape.

The nurse identifies which of the following as a potential cause of premature ventricular complexes (PVCs)? A. Hypokalemia B. Hypovolemia C. Bradycardia D. Alkalosis

A. Hypokalemia PVCs can be caused by cardiac ischemia or infarction, increased workload on the heart (eg, exercise, fever, hypervolemia, heart failure, tachycardia), digitalis toxicity, acidosis, or electrolyte imbalances, especially hypokalemia.

A patient has a persistent third-degree heart block and has had several periods of syncope. What priority treatment should the nurse anticipate for this patient? A. Insertion of a pacemaker B. Administration of epinephrine C. Insertion of an implantable cardioverter defibrillator (ICD) D. Administration of atropine

A. Insertion of a pacemaker Third-degree AV block, also known as a complete block, occurs when no atrial impulse is conducted through the AV node into the ventricles. A permanent pacemaker may be necessary if the block persists.

Which nursing intervention must a nurse perform when administering prescribed vasopressors to a client with a cardiac dysrhythmia? A. Monitor vital signs and cardiac rhythm B. Document heart rate before and after administration C. Keep the client flat for one hour after administration D. Administer every five minutes during cardiac resuscitation

A. Monitor vital signs and cardiac rhythm The nurse should monitor the client's vital signs and cardiac rhythm for effectiveness of the medication and for side effects and should always have emergency life support equipment available when caring for an acutely ill client. The side effects of vasopressor drugs are hypertension, dysrhythmias, pallor, and oliguria. It is not necessary to place a client flat during or after vasopressor administration. When administering cholinergic antagonists, documentation of the heart rate is necessary.

A client with heart failure asks the nurse how dobutamine affects the body's circulation. What is the nurse's best response? A. The medication increases the force of the myocardial contraction. B. The medication increases the heart rate. C. The medication causes the kidneys to retain fluid and increase intravascular volume. D. The medication helps the kidneys produce more urine.

A. The medication increases the force of the myocardial contraction. A positive inotropic medication increases the force of the myocardial contraction. The inotropic medication decreases heart rate; it does not cause the kidneys to retain fluid or produce more urine.

The licensed practical nurse is co-assigned with a registered nurse in the care of a client admitted to the cardiac unit with chest pain. The licensed practical nurse is assessing the accuracy of the cardiac monitor, which notes a heart rate of 34 beats/minute. The client appears anxious and states not feeling well. The licensed practical nurse confirms the monitor reading. When consulting with the registered nurse, which of the following is anticipated? A. The registered nurse administering atropine sulfate intravenously B. The registered nurse stating to administer all medications except those which are cardiotonics C. The registered nurse stating to hold all medication until the pulse rate returns to 60 beats/minute D. The registered nurse stating to administer digoxin

A. The registered nurse administering atropine sulfate intravenously The licensed practical nurse and registered nurse both identify that client's bradycardia. Atropine sulfate, a cholinergic blocking agent, is given intravenously (IV) to increase a dangerously slow heart rate. Lanoxin is not administered when the pulse rate falls under 60 beats/minute. It is dangerous to wait until the pulse rate increases without nursing intervention or administering additional medications until the imminent concern is addressed.

A client has an irregular heart rate of around 100 beats/minute and a significant pulse deficit. What component of the client's history would produce such symptoms? A. atrial fibrillation B. heart block C. atrial flutter D. bundle branch block

A. atrial fibrillation In atrial fibrillation, several areas in the right atrium initiate impulses resulting in disorganized, rapid activity. The atria quiver rather than contract, producing a pulse deficit due to irregular impulse conduction to the AV node. The ventricles respond to the atrial stimulus randomly, causing an irregular ventricular heart rate, which may be too infrequent to maintain adequate cardiac output. Atrial flutter, heart block, and bundle branch block would not produce these symptoms.

Which is not a likely origination point for cardiac dysrhythmias? A. bundle of His B. ventricles C. atria D. atrioventricular node

A. bundle of His Cardiac dysrhythmias may originate in the atria, atrioventricular node, or ventricles. They do not originate in the Bundle of His.

When the appropriate electrocardiogram (ECG) complex follows the pacing spike, it is said to be A. captured. B. triggered. C. nonsynchronous. D. inhibited.

A. captured. Capture is a term used to denote that the appropriate electrocardiogram (ECG) complex followed by the pacing spike. Triggered response means that the pacemaker will respond when it senses intrinsic heart activity. Inhibited response means that the response of the pacemaker is controlled by the activity of the client's heart.

The nurse recognizes that Premature ventricular contractions (PVCs) are considered precursors of ventricular tachycardia (VT) when they: A. occur at a rate of more than six per minute B. occur during the QRS complex C. have the same shape D. are paired with a normal beat

A. occur at a rate of more than six per minute When PVCs occur at a rate of more than six per minute, they indicate increasing ventricular irritability and are considered forerunners of VT. PVCs are dangerous when they occur on the T wave. PVCs are dangerous when they are multifocal (have different shapes). A PVC that is paired with a normal beat is termed bigeminy.

The nursing student asks the nurse to describe the difference between sinus rhythm and sinus bradycardia on the electrocardiogram strip. What is the nurse's best reply? A. "The P-R interval will be prolonged in sinus bradycardia." B. "The only difference is the heart rate." C. "The QRS complex will be smaller in sinus bradycardia." D. "The P waves will be shaped differently."

B. "The only difference is the heart rate." All characteristics of sinus bradycardia are the same as those of normal sinus rhythm except for the rate, which will be below 60 in sinus bradycardia. The P waves will be shaped differently in other dysrhythmias. The QRS is the same voltage for sinus rhythms. The P-R interval is prolonged in aterioventricular blocks.

The client asks the nurse to explain what is meant by a ventricular bigeminy cardiac rhythm. What is the best response by the nurse? A. "The heart rate is between 150 to 250 bpm." B. "The rhythm has a normal beat, then a premature beat pattern." C. "It is when the heart conduction is primarily from the aterioventricular node." D. "The rhythm is regular but fast."

B. "The rhythm has a normal beat, then a premature beat pattern." Bigeminy is a rhythm in which every other complex is a premature ventricular contraction (PVC). In trigeminy, every third complex is a PVC. The rhythm is not regular and the rate should not be 150-250 bpm.

The nurse is assigned the following client assignment on the clinical unit. For which client does the nurse anticipate cardioversion as a possible medical treatment? A. A client with third-degree heart block B. A client with atrial dysrhythmias C. A new myocardial infarction client D. A client with poor kidney perfusion

B. A client with atrial dysrhythmias he nurse is correct to identify a client with atrial dysrhythmias as a candidate for cardioversion. The goal of cardioversion is to restore the normal pacemaker of the heart, as well as, normal conduction. A client with a myocardial infarction has tissue damage. The client with poor perfusion has circulation problems. The client with heart block has an impairment in the conduction system and may require a pacemaker.

The licensed practical nurse is setting up the room for a client arriving at the emergency department with ventricular arrhythmias. The nurse is most correct to place which of the following in the room for treatment? A. An ECG machine B. A defibrillator C. A suction machine D. Cardioversion equipment

B. A defibrillator The nurse is most correct to place a defibrillator close to the client room if not in the room. The nurse realizes that clients with ventricular dysrhythmias are at a high risk for fatal heart dysrhythmia and death. A suction machine is used to remove respiratory secretions. Cardioversion is used in a planned setting for atrial dysrhythmias. An ECG machine records tracings of the heart for diagnostic purposes. Most clients with history of cardiac disorders have an ECG completed.

The nurse is working on a telemetry unit, caring for a client who develops dizziness and a second-degree heart block, Mobitz Type 1. What will be the initial nursing intervention? A. Prepare to client for cardioversion. B. Administer an IV bolus of atropine. C. Send the client to the cardiac catheterization laboratory. D. Review the client's medication record.

B. Administer an IV bolus of atropine Atropine 0.5 mg given rapidly as an intravenous bolus every 3 to 5 minutes to a maximum total dose of 3.0 mg is the medication of choice in treating symptomatic second-degree heart block. The client may need to be sent to the cardiac catheterization lab for a temporary pacemaker, but atropine should be tried first. Cardioversion is used to treat a fast heart rate. Reviewing the medication record will not help the client initially.

What is the drug of choice for a stable client with ventricular tachycardia? A. Procainamide B. Amiodarone C. Atropine D. Lidocaine

B. Amiodarone Amiodarone administered IV is the antidysrhythmic medication of choice for a stable patient with ventricular tachycardia. Atropine is used for bradycardia. Procainamide is used to treat and prevent atrial and ventricular dysrhythmias. Lidocaine is used for treating ventricular dysrhythmias.

Which dysrhythmia has an atrial rate between 250 and 400, with saw-toothed P waves? A. Ventricular tachycardia B. Atrial flutter C. Atrial fibrillation D. Ventricular fibrillation

B. Atrial flutter Atrial flutter occurs in the atrium and creates impulses at a regular atrial rate between 250 and 400 times per minute. The P waves are saw-toothed in shape. Atrial fibrillation causes a rapid, disorganized, and uncoordinated twitching of atrial musculature. Ventricular fibrillation is a rapid, disorganized ventricular rhythm that causes ineffective quivering of the ventricles. Ventricular tachycardia is defined as three or more PVCs in a row, occurring at a rate exceeding 100 beats per minute.

The nurse knows that electrocardiogram (ECG) characteristics of atrial fibrillation include what? A. Normal PR interval B. Atrial rate of 300 to 400 C. Regular rhythm D. P wave resent before each QRS

B. Atrial rate of 300 to 400 ECG characteristics of atrial fibrillation include an atrial rate of 300 to 400, a nonmeasurable PR interval, irregular rhythm, and no discernible P waves.

The nurse is providing discharge instructions to a client after a permanent pacemaker insertion. Which safety precaution will the nurse communicate to the client? A. Avoid going through airport metal detectors. B. Avoid undergoing magnetic resonance imaging (MRI). C. Never engage in activities that require vigorous arm and shoulder movement. D. Stay at least 5 feet away from microwave ovens.

B. Avoid undergoing magnetic resonance imaging (MRI). A client with a pacemaker should avoid undergoing an MRI because the magnet could disrupt pacemaker function and cause injury to the client. Disruption is less likely to occur with newer microwave ovens. The client must avoid vigorous arm and shoulder movement only for the first 6 weeks after pacemaker implantation. Airport metal detectors don't harm pacemakers; however, the client should notify airport security guards that he has a pacemaker because its metal casing and programming magnet could trigger the metal detector.

The nurse is caring for a client who has premature ventricular contractions. What sign or symptom is observed in this client? A. Fever B. Fluttering C. Hypotension D. Nausea

B. Fluttering Premature ventricular contractions usually cause a flip-flop sensation in the chest, sometimes described as "fluttering." Associated signs and symptoms include pallor, nervousness, sweating, and faintness. Symptoms of premature ventricular contractions are not nausea, hypotension, and fever.

The nurse reads an athletic client's electrocardigram. What finding will be consistent with a sinus bradycardia? A. QR interval of 0.25 seconds. B. Heart rate of 42 beats per minute (bpm). C. P-to-QR ratio of 1:2. D. PR interval of 0.24 seconds.

B. Heart rate of 42 beats per minute (bpm). The heart rate of 42 bpm is slow but normal when it occurs in athletes with a sinus bradycardia. The PR interval is prolonged at 0.24 seconds, indicating a heart block. The QR interval is prolonged and indicates ventricular delay.The ratio of P to QR should be 1:1 in sinus bradycardia.

Which rhythm is also termed a ventricular escape rhythm? A. First degree AV block B. Idioventricular rhythm C. Ventriuclar asystole D. Ventricular fibrillation

B. Idioventricular rhythm Idioventricular rhythm is also called a ventricular escape rhythm.

A nursing instructor is reviewing the parts of an EKG strip with a group of students. One student asks about the names of all the EKG cardiac complex parts. Which of the following items are considered a part of the cardiac complex on an EKG strip? Choose all that apply. A. QRT wave B. P-R interval C. T wave D. S-Q segment E. P wave

B. P-R interval C. T wave E. P wave The EKG cardiac complex waves include the P wave, the QRS complex, the T wave, and possibly the U wave. The intervals and segments include the PR interval, the ST segment, and the QT interval.

Which term is used to describe a tachycardia characterized by abrupt onset, abrupt cessation, and a QRS of normal duration? A. Atrial fibrillation B. Paroxysmal atrial tachycardia C. Sinus tachycardia D. Atrial flutter

B. Paroxysmal atrial tachycardia Paroxysmal atrial tachycardia (PAT) is often caused by a conduction problem in the AV node and is now called AV nodal reentry tachycardia. Sinus tachycardia occurs when the sinus node regularly creates an impulse at a faster-than-normal rate. Atrial flutter occurs in the atrium and creates an atrial rate between 250 to 400 times per minute. Atrial fibrillation causes a rapid, disorganized, and uncoordinated twitching of atrial musculature.

A patient has had an implantable cardioverter defibrillator inserted. What should the nurse be sure to include in the education of this patient prior to discharge? (Select all that apply.) A. The patient may have a throbbing pain that is normal B. Record events that trigger a shock sensation. C. Call for emergency assistance if feeling dizzy. D. Avoid magnetic fields such as metal detection booths. E. The patient will have to schedule monthly chest x-rays to make sure the device is patent.

B. Record events that trigger a shock sensation. C. Call for emergency assistance if feeling dizzy. D. Avoid magnetic fields such as metal detection booths. The nurse should instruct the patient to avoid large magnetic fields such as those created by magnetic resonance imaging, large motors, arc welding, electrical substations, and so forth. Magnetic fields may deactivate the device, negating its effect on a dysrhythmia. The patient should call 911 for emergency assistance if a feeling of dizziness occurs. The patient should maintain a log that records discharges of an implantable cardioverter defibrillator (ICD). Record events that precipitate the sensation of shock. This provides important data for the physician to use in readjusting the medical regimen. Throbbing pain is not normal and should be reported immediately. An initial x-ray is indicated prior to discharge, but monthly x-rays are unnecessary.

A patient with hypertension has a newly diagnosed atrial fibrillation. What medication does the nurse anticipate administering to prevent the complication of atrial thrombi? A. Atropine B. Warfarin C. Amiodarone D. Adenosine

B. Warfarin Because atrial function may be impaired for several weeks after cardioversion, warfarin is indicated for at least 4 weeks after the procedure. Patients may be given amiodarone, flecainide, ibutilide, propafenone, or sotalol prior to cardioversion to enhance the success of cardioversion and prevent relapse of the atrial fibrillation

The nurse is placing electrodes for a 12-lead electrocardiogram (ECG). The nurse would be correct in placing an electrode on which area for V1? A. Mid-clavicular line, fifth intercostal space B. Right side of sternum, fourth intercostal space C. Midway between V2 and V4 D. Left side of sternum, fourth intercostal space

B. Right side of sternum, fourth intercostal space view V1, the electrodes would be placed on the right side of the sternum, fourth intercostal space. V2 is the left side of the sternum, fourth intercostal space. V3 is midway between V2 and V4. V4 is at the mid-clavicular line, fifth intercostal space.

The nurse is teaching a beginning EKG class to staff nurses. As the nurse begins to discuss the the parts of the EKG complex, one of the students asks what the normal order of conduction through the heart is. What order does the nurse describe? A. SA node, AV node, bundle of His, the Purkinje fibers, and the right and left bundle branches B. Sinoatrial (SA) node, atrioventricular (AV) node, bundle of His, right and left bundle branches, and the Purkinje fibers C. SA node, AV node, right and left bundle branches, bundle of His, and the Purkinje fibers D. AV node, SA node, bundle of His, right and left bundle branches, and the Purkinje fibers

B. Sinoatrial (SA) node, atrioventricular (AV) node, bundle of His, right and left bundle branches, and the Purkinje fibers The correct sequence of conduction through the normal heart is the SA node, AV node, bundle of His, right and left bundle branches, and Purkinje fibers.

The client has just been diagnosed with a dysrhythmia. The client asks the nurse to explain normal sinus rhythm. What would the nurse explain is the characteristic of normal sinus rhythm? A. Impulse travels to the atrioventricular (AV) node in 0.15 to 0.5 second. B. The sinoatrial (SA) node initiates the impulse. C. The ventricles depolarize in 0.5 second or less. D. Heart rate between 60 and 150 beats/minute.

B. The sinoatrial (SA) node initiates the impulse. The characteristics of normal sinus rhythm are heart rate between 60 and 100 beats/minute, the SA node initiates the impulse, the impulse travels to the AV node in 0.12 to 0.2 second, the ventricles depolarize in 0.12 seconds or less, and each impulse occurs regularly

The nurse is caring for a client with second-degree atrioventricular block, Type I with symptomatic bradycardia. What is the most likely medication the nurse will administer? A. diltazem B. atropine sulfate C. atenolol D. nadolol

B. atropine sulfate Atropine blocks acetylcholine at parasympathetic neuroeffector sites and blocks vagal stimulation. The client will be treated with an anticholinergic that blocks the effects of the vagal nerve. Atenolol and nadolol are beta blockers that are used for chest pain, myocardial infarction, and hypertension. Diltazem is a calcium channel blocker used to treat angina or slow the heart rate.

A client reports light-headedness, chest pain, and shortness of breath. They physician orders tests to ascertain what is causing the client's problems. Which test is used to identify cardiac rhythms? A. electrocautery B. electrocardiogram C. echocardiogram D. electroencephalogram

B. electrocardiogram An electrocardiogram is used to identify normal and abnormal cardiac rhythms.

A client asks the nurse what causes the heart to be an effective pump. The nurse informs the client that this is due to the: A. inherent rhythmicity of all muscle tissue. B. inherent rhythmicity of cardiac muscle tissue. C. inherent electrons in muscle tissue. D. sufficient blood pressure.

B. inherent rhythmicity of cardiac muscle tissue. Cardiac rhythm refers to the pattern (or pace) of the heartbeat. The conduction system of the heart and the inherent rhythmicity of cardiac muscle produce a rhythm pattern, which greatly influences the heart's ability to pump blood effectively.

The nurse is instructing on home care after placement of an automatic implanted cardioverter defibrillator (AICD). Which statement, made by the client, needs clarification by the nurse? A. "I need to notify my cardiologist if I feel frequent kicks to the chest." B. "I should opt for a hand search at the airport instead of metal detector scan." C. "I need to stay away from microwaves." D. "I can continue to work with my power tools."

C. "I need to stay away from microwaves." Similar to hand tools, microwaves have shields or are grounded, making them safe for clients with AICDs. There is no restriction from microwave use. All of the other options are correct.

A nurse is teaching the client about the causes of fast heart rates. What client statement indicates the client requires more teaching? A. "I will cut back on my smoking and drinking alcohol." B. "If I take my metoprolol daily, I will be able to control my heart rate." C. "I will drink coffee with only two of my meals." D. "I will take my levothyroxine daily."

C. "I will drink coffee with only two of my meals." Stimulation of the sympathetic nervous system with caffeinated beverages, smoking, and drinking alcohol increases heart rate.The client is still drinking caffeine with two meals, increasing the risk for a fast heart rate. Taking medications such as metoprolol and levothyroxine will help the client maintain a normal heart rate by decreasing stimulation of the sympathetic nervous system.

A patient with mitral valve stenosis and coronary artery disease (CAD) is in the telemetry unit with pneumonia. The nurse assesses a 6-second rhythm strip and determines that the ventricular rhythm is highly irregular at 88, with no discernible P waves. What does the nurse determine this rhythm to be? A. Sinus tachycardia B. Nonparoxysmal junctional tachycardia C. Atrial flutter D. Ventricular flutter

C. Atrial flutter Atrial flutter occurs because of a conduction defect in the atrium and causes a rapid, regular atrial rate, usually between 250 and 400 bpm and results in P waves that are saw-toothed. Ventricular rhythm may be irregular, and P waves may be absent. Ventricular rate usually ranges between 75 and 150 bpm.

Two nursing students are reading EKG strips. One of the students asks the instructor what the P-R interval represents. The correct response should be which of the following? A. "It shows the time it takes the AV node impulse to depolarize the septum and travel through the Purkinje fibers." B. "It shows the time it takes the AV node impulse to depolarize the atria and travel through the SA node." C. "It shows the time needed for the SA node impulse to depolarize the atria and travel through the AV node." D. "It shows the time it takes the AV node impulse to depolarize the ventricles and travel through the SA node."

C. "It shows the time needed for the SA node impulse to depolarize the atria and travel through the AV node." The PR interval is measured from the beginning of the P wave to the beginning of the QRS complex and represents the time needed for sinus node stimulation, atrial depolarization, and conduction through the AV node before ventricular depolarization. In a normal heart the impulses do not travel backward. The PR interval does not include the time it take to travel through the Purkinje fibers.

The nurse receives a telephone call from a client with an implanted pacemaker who reports a pulse of 68 beats per minute, but the pacemaker rate is set at 72 beats per minute. What is the nurse's best response? A. "This is okay as long as you are not having any symptoms." B. "Try walking briskly for about 5 minutes to see if that gets your heart rate to increase." C. "Please come to the clinic right away so that we may interrogate the pacemaker to see if it is malfunctioning." D. "Don't worry. The pacemaker's rate is often higher than the client's actual heart rate."

C. "Please come to the clinic right away so that we may interrogate the pacemaker to see if it is malfunctioning." A client experiencing pacemaker malfunctioning may develop bradycardia as well as signs and symptoms of decreased cardiac output. The client should check the pulse daily and report immediately any sudden slowing or increasing of the pulse rate, which may indicate pacemaker malfunction. The client needs to be evaluated to avoid cardiac output problems. Walking will not keep the heart rate at a safe level.

A client with a second-degree atrioventricular heart block, Type II is admitted to the coronary care unit. How will the nurse explain the need to monitor the client's electrocardiogram (ECG) strip to the spouse? A. "The heart's electrical activity will be recorded when the heart rate exceeds 60 beats per minute." B. "The box is recording the heart's electrical activity, and a physician will review the tracing later." C. "The small box will transmit the heart rhythm to the central monitor all the time." D. "When your spouse needs help, an alarm will go off at the desk."

C. "The small box will transmit the heart rhythm to the central monitor all the time." In telemetry, a small box transmits the client's heart rhythm to the central unit for constant monitoring. Telemetry has nothing to do with the client needing help. A holter monitor is a device that records the heart's electrical activity and for later review by a physician. The telemetry transmits the heart rhythm regardless of the client's heart rate.

The nursing student asks the nurse how to tell the difference between ventricular tachycardia and ventricular fibrillation on an electrocardiogram strip. What is the best response? A. "The P-R interval will be prolonged in ventricular fibrillation, while in ventricular tachycardia the P-R interval is normal." B. "The two look very much alike; it is difficult to tell the difference." C. "Ventricular fibrillation is irregular with undulating waves and no QRS complex. Ventricular tachycardia is usually regular and fast, with wide QRS complexes." D. "The QRS complex in ventricular fibrillation is always narrow, while in ventricular tachycardia the QRS is of normal width."

C. "Ventricular fibrillation is irregular with undulating waves and no QRS complex. Ventricular tachycardia is usually regular and fast, with wide QRS complexes." Ventricular fibrillation is irregular with undulating waves and no QRS complex, while ventricular tachycardia is usually regular and fast with wide QRS complexes. The rhythms look different on the electrocardiogram strip. The QRS is wide and bizarre or undefined in ventricular fibrillation. The P-R interval is not present in the ventricular dysrhythmias.

The nurse knows that what PR interval presents a first-degree heart block? A. 0.16 seconds B. 0.14 seconds C. 0.24 seconds D. 0.18 seconds

C. 0.24 seconds In adults, the normal range for the PR is 0.12 to 0.20 seconds. A PR internal of 0.24 seconds would indicate a first-degree heart block.

The nurse is caring for a client who is displaying a third-degree AV block on the EKG monitor. What is the priority nursing intervention for the client? A. Assessing the client's blood pressure and heart rate frequently B. Maintaining the client's intravenous fluids C. Alerting the healthcare provider of the third-degree heart block D. Identifying the client's code level status

C. Alerting the healthcare provider of the third-degree heart block The client may experience low cardiac output with third-degree AV block. The healthcare provider needs to intervene to preserve the client's cardiac output. Monitoring the blood pressure and heart rate are important, but not the priority. The identification of a code status during a heart block is not appropriate. The IV fluids are not helpful if the heart is not perfusing.

The nurse is caring for clients on a telemetry unit. Which nursing consideration best represents concerns of altered rhythmic patterns of the heart? A. Altered patterns frequently produce neurological deficits. B. Altered patterns frequently turn into life-threatening arrhythmias. C. Altered patterns frequently affect the heart's ability to pump blood effectively. D. Altered patterns frequently cause a variety of home safety issues.

C. Altered patterns frequently affect the heart's ability to pump blood effectively. The best representation of a nursing concern related to a cardiac arrhythmia is the inability of the heart to fill the chambers and eject blow flow efficiently. Lack of an efficient method to circulate blood and bodily fluids produces a variety of complications such as tissue ischemia, pulmonary edema, hypotension, decreased urine output, and impaired level of consciousness. The other options can occur with dysrhythmias, but the cause stemming from the altered pattern is the best answer.

An operating room nurse is caring for a client who is having a pacemaker implanted. Thehealth care provider has requested a demand mode pacemaker for this client. What is this type of pacemaker? A. A temporary pacemaker B. Asynchronous C. Self-activated D. A fixed-rate pacemaker

C. Self-activated Demand (synchronous) mode pacemakers self-activate when the client's pulse falls below a certain level. A fixed-rate pacemaker is asynchronous and permanent. Temporary pacemakers are used until a permanent pacemaker can be implanted.

Which postimplantation instruction must a nurse provide to a client with a permanent pacemaker? A. Delay activities such as swimming and bowling for at least 3 weeks B. Keep moving the arm on the side where the pacemaker is inserted C. Avoid sources of electrical interference D. Keep the arm on the side of the pacemaker higher than the head

C. Avoid sources of electrical interference The nurse must instruct the client with a permanent pacemaker to avoid sources of electrical interference. The nurse should also instruct the client to avoid strenuous movement (especially of the arm on the side where the pacemaker is inserted), to keep the arm on the side of the pacemaker lower than the head except for brief moments when dressing or performing hygiene, and to delay for at least 8 weeks activities such as swimming, bowling, tennis, vacuum cleaning, carrying heavy objects, chopping wood, mowing, raking, and shoveling snow.

The nurse is observing the monitor of a patient with a first-degree atrioventricular (AV) block. What is the nurse aware characterizes this block? A. P waves hidden with the QRS complex B. An irregular rhythm C. Delayed conduction, producing a prolonged PR interval D. A variable heart rate, usually fewer than 60 bpm

C. Delayed conduction, producing a prolonged PR interval First-degree AV block occurs when all the atrial impulses are conducted through the AV node into the ventricles at a rate slower than normal. Thus the PR interval is prolonged (>0.20 seconds).

The nurse is preparing a client for upcoming electrophysiology (EP) studies and possible ablation for treatment of atrial tachycardia. What information will the nurse include in the teaching? A. After the procedure, the dysrhythmia will not recur. B. The procedure takes less time than a cardiac catheterization. C. During the procedure, the dysrhythmia will be reproduced under controlled conditions. D. The procedure will occur in the operating room under general anesthesia.

C. During the procedure, the dysrhythmia will be reproduced under controlled conditions. During EP studies, the patient is awake and may experience symptoms related to the dysrhythmia.The client does not receive general anesthesia. The EP procedure time is not easy to determine. EP studies do not always include ablation of the dysrhythmia.

The staff educator is teaching a class in dysrhythmias. What statement is correct for defibrillation? A. It is a scheduled procedure 1 to 10 days in advance. B. The client is sedated before the procedure. C. It is used to eliminate ventricular dysrhythmias. D. It uses less electrical energy than cardioversion.

C. It is used to eliminate ventricular dysrhythmias. The only treatment for a life-threatening ventricular dysrhythmia is immediate defibrillation, which has the exact same effect as cardioversion, except that defibrillation is used when there is no functional ventricular contraction. It is an emergency procedure performed during resuscitation. The client is not sedated but is unresponsive. Defibrillation uses more electrical energy (200 to 360 joules) than cardioversion.

A client asks the nurse about complications associated with use of a cardiac pacemaker. What does the nurse include in their response? Select all that apply. A. Positive Kernig's sign B. Negative Babinski reflex C. Localized infection D. Hiccuping E. Twiddler syndrome

C. Localized infection D. Hiccuping E. Twiddler syndrome Complications associated with pacemakers include infection at entry site, pneumothorax, bleeding and hematoma, hemothorax, ventricular ectopy and tachycardia, phrenic nerve/diaphragmatic(hiccupping)/skeletal stimulation, cardiac perforation, Twiddler syndrome, and hemodynamic instability. A positive Kernig's sign is an indication of meningitis. A positive Babinski reflex is normal in neonates, but indicates a central nervous system disorder in adults.

To evaluate a client's atrial depolarization, the nurse observes which part of the electrocardiogram waveform? A. T wave B. QRS complex C. P wave D. PR interval

C. P wave The P wave depicts atrial depolarization, or spread of the electrical impulse from the sinoatrial node through the atria. The PR interval represents spread of the impulse through the interatrial and internodal fibers, atrioventricular node, bundle of His, and Purkinje fibers. The QRS complex represents ventricular depolarization. The T wave depicts the relative refractory period, representing ventricular repolarization.

A healthy adult client is seeing a health care provider for an annual physical examination. While the nurse is taking the client's vital signs, the client states, "Occasionally, my heart skips a beat. Is this normal?" What is the nurse's best response? A. Atrial flutter B. Ventricular fibrillation C. Premature atrial complex D. Sinus tachycardia

C. Premature atrial complex A premature atrial complex (PAC) is a single ECG complex that occurs when an electrical impulse starts in the atrium before the next normal impulse of the sinus node.

The nurse analayzes the electrocardiogram (ECG) tracing of a client newly admitted to the cardiac step-down unit with a diagnosis of chest pain. Which finding indicates the need for follow-up? A. ST segment that is isoelectric in appearance B. PR interval that is 0.18 seconds long C. QT interval that is 0. 46 seconds long D. QRS complex that is 0.10 seconds long

C. QT interval that is 0. 46 seconds long The QT interval that is 0.46 seconds long needs to be investigated. The QT interval is usually 0.32 to 0.40 seconds in duration if the heart rate is 65 to 95 bpm. If the QT interval becomes prolonged, the client may be at risk for a lethal ventricular dysrhythmia, called torsades de pointes. The other findings are normal.

The nurse is proving discharge instructions for a client with a new arrhythmia. Which statement should the nurse include? A. It is not necessary to learn how to take your own pulse. B. If you miss a dose of your antiarrhythmia medication, double up on the next dose. C. Your family and friends may want to take a CPR class. D. Do not be concerned if you experience symptoms of lightheadedness and dizziness.

C. Your family and friends may want to take a CPR class. Having friends and family learn to perform CPR will help the client manage the arrhythmia. Monitoring pulse rate at home also helps the client manage the condition. Antiarrhythmic medication should be taken on time. Lightheadedness and dizziness should be reported to the provider.

The client has been prescribed procainamide for a dysrhythmia. Which medication side effect will the nurse teach the client to watch for? A. hypertension B. tachycardia C. feeling tired D. change in mental status

C. feeling tired The side effects of procainamide hydrochloride can include hypotension, GI upset, and feelings of tiredness. Procainamide does not cause hypertension, tachycardia, or a change in mental status.

The nurse assesses a client with a heart rate of 120 beats per minute. What are the known causes of sinus tachycardia? A. vagal stimulation B. digoxin C. hypovolemia D. hypothyroidism

C. hypovolemia The causes of sinus tachycardia include physiologic or psychological stress (acute blood loss, anemia, shock, hypovolemia, fever, and exercise). Vagal stimulation, hypothyroidism, and digoxin will cause a sinus bradycardia.

The client returns to the clinic for a follow-up appointment following a permanent pacemaker insertion and reports tenderness and throbbing around the incision. The nurse observes mild swelling, erythema, and warmth at the pacemaker insertion site. What does the nurse suspect? A. normal postoperative healing B. internal bleeding at pacemaker site C. pacemaker site infection D. postoperative site hematoma

C. pacemaker site infection Postoperative care for a pacemaker insertion includes observing for symptoms of infection. These symptoms include swelling, unusual tenderness, drainage, and increased warmth. When the site is healing normally, there will be no tenderness and throbbing. A hematoma forms a lump at the pacemaker insertion site. Mild bleeding will be drainage.

A client is diagnosed with a dysrhythmia at a rate slower than 60 beats/minute. What type of dysrhythmia does the client have? A. atrial bradycardia B. heart block C. sinus bradycardia D. none

C. sinus bradycardia Sinus bradycardia is a dysrhythmia that proceeds normally through the conduction pathway but at a slower than usual (less than60 beats/minute) rate.

When the nurse observes that the client's heart rate increases during inspiration and decreases during expiration, the nurse reports that the client is demonstrating A. sinus tachycardia. B. sinus bradycardia. C. sinus dysrhythmia. D. normal sinus rhythm.

C. sinus dysrhythmia. Sinus dysrhythmia occurs when the sinus node creates an impulse at an irregular rhythm. Normal sinus rhythm occurs when the electrical impulse starts at a regular rate and rhythm in the SA node and travels through the normal conduction pathway. Sinus bradycardia occurs when the sinus node regularly creates an impulse at a slower-than-normal rate. Sinus tachycardia occurs when the sinus node regularly creates an impulse at a faster-than-normal rate.

The nurse is caring for a 32-year-old client admitted with a medical diagnosis of atrial fibrillation, related to "holiday heart" syndrome. A nursing student working with the nurse asks for information about "holiday heart" syndrome. Which is the best response by the nurse? A. "This is the association of heart dysrhythmias, especially atrial fibrillation, with physical activity the client is not used to." B. "This is the association of heart dysrhythmias, especially atrial fibrillation, with very heavy meals." C. "This is the association of heart dysrhythmias, especially atrial fibrillation, with extramarital sex." D. "This is the association of heart dysrhythmias, especially atrial fibrillation, with binge drinking."

D. "This is the association of heart dysrhythmias, especially atrial fibrillation, with binge drinking." Atrial fibrillation may be found in people with acute moderate to heavy ingestion of alcohol.

The nurse is caring for a client who had a permanent pacemaker surgically placed and is now ready for discharge. What statement made by the client indicates the need for more education? A. "I will check my pulse every day and report to the doctor if the rate is below the pacemaker setting." B. "I will avoid any large magnets that may affect my pacemaker." C. "I will call the doctor if my incision becomes swollen and red." D. "We will be getting rid of our microwave oven so it will not affect my pacemaker."

D. "We will be getting rid of our microwave oven so it will not affect my pacemaker." Permanent pacemaker generators have filters that protect them from electrical interference from most household devices, motors, and appliances, so the client can keep the microwave oven. Clients are taught to check pulses daily, avoid large magnets, and report any incisional redness or swelling.

A client has been living with an internal, fixed-rate pacemaker. When checking the client's readings on a cardiac monitor the nurse notices an absence of spikes. What should the nurse do? A. Measure the client's blood pressure. B. Suggest the need for a new beta-blocker to the doctor. C. Do nothing; there is no cause for alarm. D. Double-check the monitoring equipment.

D. Double-check the monitoring equipment. One of the reasons for lack of pacemaker spikes is faulty monitoring equipment.

A home care nurse is visiting a left-handed client who has an implantable cardioverter-defibrillator (ICD) implanted in the left chest. The client is planning to go rifle hunting. How should the nurse respond? A. "You'll need to take an extra dose of your antiarrhythmic before you shoot." B. "Being that close to a rifle might make your ICD fire." C. "Enjoy your hunting trip." D. "You can't shoot a rifle left-handed because the rifle's recoil will traumatize the ICD site."

D. "You can't shoot a rifle left-handed because the rifle's recoil will traumatize the ICD site." The recoil from the rifle can damage the ICD, so the client should be warned against shooting a rifle with the left hand. Close proximity to a rifle won't cause the ICD to fire inadvertently. The client shouldn't take an extra dose of an antiarrhythmic.

The nurse is working on a telemetry unit. When the nurse is interpreting a client's heart rhythm, the nurse counts each large block on graph paper as how many seconds? A. 0.4 B. 0.3 C. 0.1 D. 0.2

D. 0.2 Each small block on the graph paper equals 0.04 second, and five small blocks form a large block, which equals 0.2 second.

The nurse cares for a client with a dysrhythmia and understands that the P wave on an electrocardiogram (ECG) represents which phase of the cardiac cycle? A. Early ventricular repolarization B. Ventricular depolarization C. Ventricular repolarization D. Atrial depolarization

D. Atrial depolarization The P wave represents atrial depolarization. The QRS complex represents ventricular depolarization. The T wave represents ventricular repolarization. The ST segment represents early ventricular repolarization, and lasts from the end of the QRS complex to the beginning of the T wave.

A client's electrocardiogram (ECG) tracing reveals a ventricular rate between 250 and 400, with saw-toothed P waves. The nurse correctly identifies this dysrhythmia as A. Atrial fibrillation B. Ventricular tachycardia C. Ventricular fibrillation D. Atrial flutter

D. Atrial flutter The nurse correctly identifies the electrocardiogram (ECG) tracing as atrial flutter. Atrial flutter occurs in the atrium and creates impulses at a regular atrial rate between 250 and 400 times per minute. The P waves are saw-toothed in appearance. Atrial fibrillation causes a rapid, disorganized, and uncoordinated twitching of atrial musculature. The atrial rate is 300 to 600, and the ventricular rate is usually 120 to 200 in untreated atrial fibrillation. There are no discernible P waves. Ventricular fibrillation is a rapid, disorganized ventricular rhythm that causes ineffective quivering of the ventricles. The ventricular rate is greater than 300 per minute and extremely irregular, without a specific pattern. The QRS shape and duration is irregular, undulating waves without recognizable QRS complexes. Ventricular tachycardia is defined as three or more PVCs in a row, occurring at a rate exceeding 100 beats per minute.

The licensed practical nurse is monitoring the waveform pattern on the cardiac monitor ofthe client admitted following a myocardial infarction. The nurse notes that every other beat includes a premature ventricular contraction (PVC). The nurse notes which of the following in the permanent record? A. R-on-T phenomenon B. Multifocal PVCs C. Couplets D. Bigeminy

D. Bigeminy The nurse is correct to note bigeminy on the permanent record when every other beat is a PVC. Couplets are two PVCs in a row. Multifocal PVCs originate from more than one location. R-on-T phenomenon occurs when the R wave falls on the T wave.

A nurse is caring for a client with a history of cardiac disease and type 2 diabetes. The nurse is closely monitoring the client's blood glucose level. Which medication is the client most likely taking? A. Procainamide B. Diltiazem C. Amiodarone D. Carvedilol

D. Carvedilol The nurse must monitor blood glucose levels closely in clients with type 2 diabetes who are taking beta-adrenergic blockers such as carvedilol, because beta-adrenergic blockers may mask the signs of hypoglycemia. The nurse should monitor QRS duration in clients taking procainamide and pulmonary function in clients taking amiodarone (because the drug may cause pulmonary fibrosis). Diltiazem may cause an increased PR interval or bradycardia.

A nurse is caring for a client who has been admitted to have a cardioverter defibrillator implanted. The nurse knows that implanted cardioverter defibrillators are used in which clients? A. Clients with recurrent life-threatening bradycardias B. Clients with sinus tachycardia C. Clients with ventricular bradycardia D. Clients with recurrent life-threatening tachydysrhythmias

D. Clients with recurrent life-threatening tachydysrhythmias The automatic implanted cardioverter defibrillator (AICD) is an internal electrical device used for selected clients with recurrent life-threatening tachydysrhythmias. Therefore, options A, B, and C are incorrect.

After evaluating a client for hypertension, a health care provider orders atenolol, 50 mg P.O. daily. Which therapeutic effect should atenolol have in treating hypertension? A. Decreased peripheral vascular resistance B. Increased cardiac output and increased systolic and diastolic blood pressure C. Decreased blood pressure with reflex tachycardia D. Decreased cardiac output and decreased systolic and diastolic blood pressure

D. Decreased cardiac output and decreased systolic and diastolic blood pressure As a long-acting, selective beta1-adrenergic blocker, atenolol decreases cardiac output and systolic and diastolic blood pressure; however, like other beta-adrenergic blockers, it increases peripheral vascular resistance at rest and with exercise. Atenolol may cause bradycardia, not tachycardia.

A nurse is caring for a client with atrial fibrillation. What procedure would the nurse educate the patient about for termination of the dysrhythmia? A. Pacemaker implantation B. Defibrillation C. Mace procedure D. Elective cardioversion

D. Elective cardioversion Atrial fibrillation also is treated with elective cardioversion or digitalis if the ventricular rate is not too slow. Defibrillation is used for a ventricular problem. A Mace procedure is only a distractor for this question. Pacemakers are implanted for bradycardia.

Which nursing intervention is required to prepare a client with cardiac dysrhythmia for an elective electrical cardioversion? A. Administer digitalis and diuretics 24 hours before cardioversion B. Monitor blood pressure every 4 hours C. Facilitate CPR until the client is prepared for cardioversion D. Instruct the client to restrict food and oral intake

D. Instruct the client to restrict food and oral intake The nurse should instruct the client to restrict food and oral intake before the cardioversion procedure. Digitalis and diuretics are withheld for 24 to 72 hours before cardioversion. The presence of digitalis and diuretics in myocardial cells decreases the ability to restore normal conduction and increases the chances of a fatal dysrhythmia developing after cardioversion. When the client is in cardiopulmonary arrest, the nurse should facilitate CPR until the client is prepared for defibrillation and not for cardioversion. Monitoring blood pressure every 4 hours is not required to prepare a client with cardiac dysrhythmia.

A client admitted to the telemetry unit has a serum potassium level of 6.6 mEq/L. Which electrocardiographic (ECG) characteristic is commonly associated with this laboratory finding? A. Prolonged QT interval B. Occasional U waves C. Flattened P waves D. Peaked T waves

D. Peaked T waves The client's serum potassium level is high. The T wave is an ECG characteristic reflecting repolarization of the ventricles. It may become tall or "peaked" if a client's serum potassium level is high. The U wave is an ECG waveform characteristic that may reflect Purkinje fiber repolarization. It is usually seen when a client's serum potassium level is low. The P wave is an ECG characteristic reflecting conduction of an electrical impulse through the atria and is not affected by a client's serum potassium level. The QT interval is an ECG characteristic reflecting the time from ventricular depolarization to repolarization, and is not affected by a client's serum potassium level.

The nurse observes an electrocardiogram (ECG) tracing on a cardiac monitor with a pattern in lead II as well as a bizarre, abnormal shape to the QRS complex. The nurse has likely observed which ventricular dysrhythmia? A. Ventricular bigeminy B. Ventricular fibrillation C. Ventricular tachycardia D. Premature ventricular contraction

D. Premature ventricular contraction A premature ventricular contraction (PVC) is an impulse that starts in a ventricle before the next normal sinus impulse. Ventricular bigeminy is a rhythm in which every other complex is a PVC. Ventricular tachycardia is defined as three or more PVCs in a row, occurring at a rate exceeding 100 beats per minute. Ventricular fibrillation is a rapid but disorganized ventricular rhythm that causes ineffective quivering of the ventricles.

Which ECG waveform characterizes conduction of an electrical impulse through the left ventricle? A. P wave B. QT interval C. PR interval D. QRS complex

D. QRS complex The QRS complex represents ventricular depolarization. The P wave is an ECG characteristic reflecting conduction of an electrical impulse through the atria. The PR interval is a component of an ECG tracing reflecting conduction of an electrical impulse through the AV node. The QT interval is an ECG characteristic reflecting the time from ventricular depolarization to repolarization.

A 28-year-old client presents to the emergency department, stating severe restlessness and anxiety. Upon assessment, the client's heart rate is 118 bpm and regular, the client's pupils are dilated, and the client appears excitable. Which action should the nurse take next? A. Instruct the client to hold the breath and bear down. B. Place the client on supplemental oxygen. C. Prepare to administer a calcium channel blocker. D. Question the client about alcohol and illicit drug use.

D. Question the client about alcohol and illicit drug use. The client is experiencing sinus tachycardia. Since the client's findings of tachycardia, dilated pupils, restlessness, anxiety, and excitability can indicate illicit drug use (cocaine), the nurse should question the client about alcohol and illicit drug use. This information will direct the client's plan of care. Causes of tachycardia include medications that stimulate the sympathetic response, stimulants, and illicit drugs. The treatment goals for sinus tachycardia is usually determined by the severity of symptoms and directed at identifying and abolishing its cause. The other interventions may be implemented, but determining the cause of the tachycardia is essential.

When no atrial impulse is conducted through the AV node into the ventricles, the client is said to be experiencing which type of AV block? A. Second degree, type II B. First degree C. Second degree, type I D. Third degree

D. Third degree In third degree heart block, two impulses stimulate the heart, one impulse stimulates the ventricles and other stimulates the atria. In first degree heart block, all the atrial impulses are conducted through the AV node into the ventricles at a rate slower than normal. In second degree AV block, type I, all but one of the atrial impulses are conducted through the AV node into the ventricles. In second degree AV block, type II, only some of the atrial impulses are conducted through the AV node into the ventricles.

The nurse participates in the care of a client requiring emergent defibrillation. The nurse determines the steps should be completed in which order? A. Charge the defibrillator to the prescribed voltage. B. Apply the multifunction conductor pads to the client's chest. C. Deliver the prescribed electrical charge. D. Turn on the defibrillator and place it in "not sync" mode. E. Call "clear" three times ensuring client and environmental safety.

D. Turn on the defibrillator and place it in "not sync" mode. A. Charge the defibrillator to the prescribed voltage. B. Apply the multifunction conductor pads to the client's chest. E. Call "clear" three times ensuring client and environmental safety. C. Deliver the prescribed electrical charge. This is the sequence of events the nurse should implement when delivering emergent defibrillation. If not followed correctly, the client and health care team may be placed in danger.

The nurse is admitting a client to a telemetry unit with an atrial dysrhythmia. What symptoms will the nurse further evaluate? A. leg pain B. hypertension C. hypocarbia D. chest pain

D. chest pain Clients with atrial dysrhythmias may have chest pain, shortness of breath, and low blood pressure. Leg pain is not common with atrial dysrhythmias. Hypocarbia is seen with reduced carbon dioxide, not common with chrpulmonary disease.

The nurse analyzes a 6-second electrocardiogram (ECG) tracing. The P waves and QRS complexes are regular. The PR interval is 0.18 seconds long, and the QRS complexes are 0.08 seconds long. The heart rate is calculated at 70 bpm. The nurse correctly identifies this rhythm as A. junctional tachycardia. B. first-degree atrioventricular block. C. sinus tachycardia. D. normal sinus rhythm.

D. normal sinus rhythm. The electrocardiogram (ECG) tracing shows normal sinus rhythm (NSR). NSR has the following characteristics: ventricular and atrial rate: 60 to 100 beats per minute (bpm) in the adult; ventricular and atrial rhythm: regular; and QRS shape and duration: usually normal, but may be regularly abnormal; P wave: normal and consistent shape, always in front of the QRS; PR interval: consistent interval between 0.12 and 0.20 seconds and P:QRS ratio: 1:1.

A client has a heart rate greater than 155 beats/minute and the ECG shows a regular rhythm with a rate of 162 beats/minute. The client is intermittently alert and reports chest pain. P waves cannot be identified. What condition would the nurse expect the physician to diagnose? A. atrial flutter B. sinus tachycardia C. heart block D. supraventricular tachycardia

D. supraventricular tachycardia Supraventricular tachycardia (SVT) is a dysrhythmia in which the heart rate has a consistent rhythm but beats at a dangerously high rate (over 150 beats/minute). P waves cannot be identified on the ECG. Diastole is shortened and the heart does not have sufficient time to fill. These symptoms do not suggest sinus tachycardia, heart block, or atrial flutter.

The nurse working in the emergency department places a client in anaphylactic shock on a cardiac monitor and sees the cardiac rhythm below. What dysthymia should the nurse document? (Rhythm of qrs complexes that are very disorganized) A. atrial flutter B. sinus bradycardia C. idioventricular rhythm D. ventricular fibrillation E. sinus rhythm with third-degree atrioventricular block

D. ventricular fibrillation he dysrhythmia is ventricular fibrillation because it is a rapid rhythm with no organization. This dysrhythmia causes ventricles to quiver and there is no atrial activity on the ECG. Atrial flutter will have atrial rate ranges between 250 and 400 bpm, ventricular rate between 75 and 150 bpm. The QRS shape and duration are usually normal, but may be abnormal or absent. The P-wave will be saw-toothed in shape. The PR-interval may be difficult to determine. Sinus Bradycardia will have a ventricular and atrial rate of less than 60 bpm. The ventricular and atrial rhythm will be regular. The QRS shape and duration is usually normal, but may be regularly abnormal. The P-wave will be normal and consistent in shape. The PR interval will have an interval between 0.12 and 0.20 seconds. Idioventricular rhythm will have a ventricular rate between 20 and 40 bpm. The ventricular rhythm will be regular. The QRS shape and duration will be abnormal shape, bizarre, and will have a duration of 0.12 seconds or more. Sinus rhythm with third-degree atrioventricular block will have a ventricular and atrial rhythm with a PP interval that is regular and the RR interval that is regular. The PP interval is not equal to the RR interval. The QRS shape and duration will depend on the escape rhythm. With junctional rhythm, the QRS shape and duration are usually normal; with an idioventricular rhythm, the QRS shape and duration are usually abnormal. The P-wave will depend on the underlying rhythm. The PR interval is very irregular. The P:QRS ratio will show more P-waves than QRS complexes.

A nurse provides evening care for a client wearing a continuous telemetry monitor. While the nurse is giving the client a back rub, the client 's monitor alarm sounds and the nurse notes a flat line on the bedside monitor system. What is the nurse's first response? A. Call a code and obtain the crash cart. B. Call for assistance and begin CPR. C. Administer a pericardial thump. D.Assess the client and monitor leads.

D.Assess the client and monitor leads. The nurse should assess the client and monitor leads first. It is important that the nurse "treat the client, not the monitor." Ventricular asystole may often appear on the monitor when leads are displaced. The other interventions are not necessary.


Conjuntos de estudio relacionados

Vander's Ch 11: Endocrine System

View Set

Chapter 7: Life Span Development

View Set

Stroke, Intracranial Pressure, Increased Intracranial Pressure, HIV, Conflict Resolution

View Set

Personal Finance Investing Unit Review Part 2

View Set

Policy Provisions and Contract Law

View Set

History Section 3 Review/ Chapter 15

View Set